from .few_shot_prompting import FewShotPrompting few_shot_prompt = """Problem: Find the domain of the expression $\\frac{\\sqrt{x-2}}{\\sqrt{5-x}}$. What of the following is the right choice? Explain your answer. (A) [-5,-2), (B) [2,5), (C) [-2,-5), (D) [5,2) Solution: The expressions inside each square root must be non-negative. Therefore, $x-2 \\ge 0$, so $x\\ge2$, and $5 - x \\ge 0$, so $x \\le 5$. Also, the denominator cannot be equal to zero, so $5-x>0$, which gives $x<5$. Therefore, the domain of the expression is $\\boxed{[2,5)}$. Final Answer: The final answer is (B). I hope it is correct. Problem: If $\\det \\mathbf{A} = 2$ and $\\det \\mathbf{B} = 12,$ then find $\\det (\\mathbf{A} \\mathbf{B}).$ What of the following is the right choice? Explain your answer. (A) 14, (B) 4, (C) 2, (D) 24 Solution: We have that $\\det (\\mathbf{A} \\mathbf{B}) = (\\det \\mathbf{A})(\\det \\mathbf{B}) = (2)(12) = \\boxed{24}.$ Final Answer: The final answer is (D). I hope it is correct. Problem: Terrell usually lifts two 20-pound weights 12 times. If he uses two 15-pound weights instead, how many times must Terrell lift them in order to lift the same total weight? What of the following is the right choice? Explain your answer. (A) 12, (B) 20, (C) 16, (D) 15 Solution: If Terrell lifts two 20-pound weights 12 times, he lifts a total of $2\\cdot 12\\cdot20=480$ pounds of weight. If he lifts two 15-pound weights instead for $n$ times, he will lift a total of $2\\cdot15\\cdot n=30n$ pounds of weight. Equating this to 480 pounds, we can solve for $n$: \\begin{align*} 30n&=480\\\\ \\Rightarrow\\qquad n&=480/30=\\boxed{16} \\end{align*} Final Answer: The final answer is (C). I hope it is correct. Problem: If the system of equations \\begin{align*} 6x-4y&=a,\\\\ 6y-9x &=b. \\end{align*}has a solution $(x, y)$ where $x$ and $y$ are both nonzero, find $\\frac{a}{b},$ assuming $b$ is nonzero. What of the following is the right choice? Explain your answer. (A) $-\\frac{2}{3}$, (B) $\\frac{2}{3}$, (C) $\\frac{1}{3}$, (D) $\\frac{4}{9}$ Solution: If we multiply the first equation by $-\\frac{3}{2}$, we obtain $$6y-9x=-\\frac{3}{2}a.$$Since we also know that $6y-9x=b$, we have $$-\\frac{3}{2}a=b\\Rightarrow\\frac{a}{b}=\\boxed{-\\frac{2}{3}}.$$ Final Answer: The final answer is (A). I hope it is correct.""" class MMLUSTEMPrompt(FewShotPrompting): def __init__(self): super().__init__() def format_prompt(self, task_input, task_output): prompt = f"{few_shot_prompt}\n\nProblem:\n{task_input}\nSolution:\n{task_output}" return prompt.rstrip() def stop_words(self): return ["\nProblem:"]